A trig problem for your entertainment

The Rocketry Forum

Help Support The Rocketry Forum:

This site may earn a commission from merchant affiliate links, including eBay, Amazon, and others.

neil_w

OpenRocketeer
TRF Supporter
Joined
Jul 14, 2015
Messages
16,701
Reaction score
11,541
Location
Northern NJ
Given the following figure, what is theta? I feel like I should be able to get this but my trig skills are failing me. Please show your work, so I can understand the solution.
1638914512203.png
 
1638915343631.png

I mean, you're dealing with right angle triangles... Since the total of all angles added together is 180 degrees, you've got a known angle (90), now you just need to pick some lengths and using the math you should be able to compute the other two angles.
 
Last edited:
Does this work? Looking at the drawing sideways.
At theta = 0 the delta y is 1.283 - 0.991 = 0.292,
Then at theta = ?, x=3.0 and y=.292
Resulting in arcTAN(theta) = .292 / 3.0
Theta = 5.56 deg
 
You used "Trig" and "Entertainment" in a sentence without the important linking phrase "is not".

At times like these I draw it in Inkscape and use the 'ruler' tool which gives me the angle.
I was looking for a small protractor in my desk at work, but we don't have one out here.
 
Just look at the right-most triangle and use tg.
 
Hyp of rect (h) = sqrt(3^2 + 1.283^2) = 3.262834504

If you draw a circle from the bottom right corner of the rectangle with radius (r) 0.991 and then take a tangent line from that circle to the top left corner - you don't need to know the length of this line as we already have length of the hyp of the rectangle and radius of the circle. So, we can gather the angle of that tangent line and the hyp: arcsin(r/h) = 0.308598507 radians or 17.68139202 deg
We can gather the hyp angle of the rectangle by arccos(long side/h) = arccos(3/3.262834504) = 0.404127168 radians or 23.15478112 deg
Now the easy part - subtract the previous angle from the hyp angle = 0.404127168 - 0.308598507 = 0.095528661 radians or 5.473389103 deg

Brian S was correct.

TP
 
Thank you for all the answers so far. I should have clarified up front that I am looking for an analytical solution. Thank you K'Tesh for point out that showing theta in the other locations would have made for a much simpler and clearer drawing, no idea why I didn't do that in the first place.
Does this work? Looking at the drawing sideways.
At theta = 0 the delta y is 1.283 - 0.991 = 0.292,
Then at theta = ?, x=3.0 and y=.292
Resulting in arcTAN(theta) = .292 / 3.0
Theta = 5.56 deg
Given the small angle, it works well as a very easy-to-calculate approximation, and just saying "around 5.5 degrees" would be more than enough accuracy for my needs, but it is not a precise solution.
5.47339°
or
0.0955287 radians
or
5°28'24"
Did you solve this analytically or numerically or empirically? If analytically, can you describe how you did it?
Hyp of rect (h) = sqrt(3^2 + 1.283^2) = 3.262834504

If you draw a circle from the bottom right corner of the rectangle with radius (r) 0.991 and then take a tangent line from that circle to the top left corner - you don't need to know the length of this line as we already have length of the hyp of the rectangle and radius of the circle. So, we can gather the angle of that tangent line and the hyp: arcsin(r/h) = 0.308598507 radians or 17.68139202 deg
We can gather the hyp angle of the rectangle by arccos(long side/h) = arccos(3/3.262834504) = 0.404127168 radians or 23.15478112 deg
Now the easy part - subtract the previous angle from the hyp angle = 0.404127168 - 0.308598507 = 0.095528661 radians or 5.473389103 deg
I apologize but I'm having trouble understanding exactly where you're drawing the circle and the tangent, and without that I can't follow the rest. If you could provide a simple picture it would be a big help.
 
trig1.png

hypotenuse - pink
hypotenuse angle - brown
Tangent Line (dotted blue line from circle tangent to top left corner of box)
tangent line <> hypotenuse angle - light grey
Circle Radius - cyan
theta (answer) - white

Hope that helps,

TP
 
Thank you for all the answers so far. I should have clarified up front that I am looking for an analytical solution. Thank you K'Tesh for point out that showing theta in the other locations would have made for a much simpler and clearer drawing, no idea why I didn't do that in the first place.

Given the small angle, it works well as a very easy-to-calculate approximation, and just saying "around 5.5 degrees" would be more than enough accuracy for my needs, but it is not a precise solution.

Did you solve this analytically or numerically or empirically? If analytically, can you describe how you did it?

I apologize but I'm having trouble understanding exactly where you're drawing the circle and the tangent, and without that I can't follow the rest. If you could provide a simple picture it would be a big help.

Maybe it was his wording. He said to draw a circle that was R0.991" from the lower right corner and the left red line (he posted a drawing right before me. Red in your sketch, blue dotted in his sketch) was tangent to that (i.e. its a right angle, so right triangle math works). You could skip drawing the circle and just use the exact same arcsin he said if you saw the triangle. I do draw circles myself, as sometimes seeing the tangent reminds me that you can use either rectangular coordinates (Pythagorean theorem) or polar coordinates. Its all the same, but sometimes seeing the circle can let you skip a step or two (or at least keep you on track without jumbling math).

Sandy.
 
hypotenuse - pink
hypotenuse angle - brown
Tangent Line (dotted blue line from circle tangent to top left corner of box)
tangent line <> hypotenuse angle - light grey
Circle Radius - cyan
theta (answer) - white
Thanks. On further question, though... in your picture the angle t is shown between the hypotenuse and the blue dotted line. On the one hand, you say the blue line is the tangent line that intersects in the upper left corner... but as drawn it does not. It is one of the sides of the canted rectangle, whose upper left corner is not the same as the the one where the hypotenuse terminates. The blue dotted line is correct as drawn, because that properly defines theta.

And so therefore the angle of the theta + t is not quite equal to the angle of the hypotenuse (it's very close, but not exact.)

Apologies if I am misunderstanding what you wrote (or drew), but I can't figure out any other way to interpret it.
 
Let me change that dark blue dotted line to a nice bright solid yellow one:

trig1b.png
Zoomed in:
trig1c.png

note: if that line wasn't extending to the upper corner of the rectangle, then I couldn't really utilise the hypotenuse of the rectangle.

Of course, as it turns out, it passes through the upper left corners of both rectangles.

TP
 
Ah, relooking at your original diagram, I see what you're saying, *Yours* doesn't pass through it.... Hummm... that could be a problem. Lemme ponder...

Is your diagram suppose to illustrate it passing through it? If it doesn't pass through it, we really need the distance it misses by.


Scrub that, it must be passing through it. Otherwise the 0.991 dimension would be meaningless. Just one of those inexact raster type drawings where lines are drawn close enough for purpose, but not exact.

TP
 
Last edited:

Using K'Tesh's drawing highlighting the alternate angles = Theta. Specifically the one on far right.

Side opposite Theta = (1.283-0.991)" = 0.292" = opp

Side adjacent Theta = 3" = adj

Tan Theta = opp/adj

Theta = arctan(opposite/adj) = arctan(0.292/3) = arctan(0.09733333333)

Theta = 5.55927755238 degrees approximately only due to the repeating "3" beginning in the 1/10,000ths place. The answer has three reliable digits to right of decimal, so rounding from there to hundredths seems accurate (enough) to me. :)

Thus, Theta is approximately 5.56 degrees.

Pardon the formatting.... I did this in bed, in the dark, at 0500hrs on my phone. ;)

Oops, I did not read far enough.... KevinT used the same geometry/trig. that I did, and of course gave you the answer first. Well done Kevin.

Thank you K'Tesh for the use of your concise diagram.

A long, LONG time since I did that problem.... 1981. Thank you W.J. Harlan for being such an outstanding teacher.
 
Last edited:
Ah, relooking at your original diagram, I see what you're saying, *Yours* doesn't pass through it.... Hummm... that could be a problem. Lemme ponder...

Is your diagram suppose to illustrate it passing through it? If it doesn't pass through it, we really need the distance it misses by.


Scrub that, it must be passing through it. Otherwise the 0.991 dimension would be meaningless. Just one of those inexact raster type drawings where lines are drawn close enough for purpose, but not exact.
I had to go back and look at our respective drawings more carefully to figure out the discrepancy. One aspect of your drawing threw me off, but now I realize it was a result of my initial bad choice on where to show theta (rather than the more natural location that K'Tesh showed).

So as far as I'm concerned your solution is correct. 👍 Thanks for talking me through it. Thanks also to @StreuB1 for the correct answer.

Now I have to look at @cautery and @KevinT 's answers in more detail and see why they're getting a different result.
 
I had to go back and look at our respective drawings more carefully to figure out the discrepancy. One aspect of your drawing threw me off, but now I realize it was a result of my initial bad choice on where to show theta (rather than the more natural location that K'Tesh showed).

So as far as I'm concerned your solution is correct. 👍 Thanks for talking me through it. Thanks also to @StreuB1 for the correct answer.

Now I have to look at @cautery and @KevinT 's answers in more detail and see why they're getting a different result.

Maybe I can clarify our reasoning without mucking up the water too much.

First, I am operating on a number of assumptions that were not clarified in the original problem....

1) lines bounding 1.283" and 0.991" dimensions respectively are parallel to each other.
2) left boundary lines on both dimensions intersect the same point at the top there....
3) other...?

Thus, the side opposite Theta becomes a simple line segment difference problem....

The side adjacent to Theta is explicitly called out as 3".

By definition, the tangent of any acute angle in a right triangle is equal to the length of the opposite side divided by the length of the adjacent side.

This is a simple ratio problem, but results in an approximate result due to the repeating sequence beginning in the 4th place to the right of the decimal.

But taking the arctan on the extended approximation preserves the 3 digit significance limit created by the 0.292 numerator in the ratio.

The PROPER logic to use in rounding the final answer approximation is to round down to three significant digits.... My logic while giving the correct approximation was more intuition than logoc.

Sorry, I 'be been out of the everyday trig game for a long time. ;)
 
Last edited:
First, I am operating on a number of assumptions that were not clarified in the original problem....

1) lines bounding 1.283" and 0.991" dimensions respectively are parallel to each other.
2) left boundary lines on both dimensions intersect the same point at the top there....
3) other...?
Assumptions are correct. Here, however, appears to be the error:
1638976680708.png
1638976426633.png
The black segment at the bottom left or top right has length 0.292*cos(theta), which comes out slightly less than 0.292.

Actually: 1.283 - (.991*cos(theta)), slightly larger than 0.292.
 
Last edited:
Assumptions are correct. Here, however, appears to be the error:
View attachment 493079
View attachment 493078
The black segment at the bottom left or top right has length 0.292*cos(theta), which comes out slightly less than 0.292.
I am out of town, so I don't have the ability to draw my "proof", and I am not inclined to argue over 9/100ths of 1 degree.
Sufficient to say that I disagree... ;)

Sure was fun exercising the brain....

Curious, source and purpose of the problem?
 
The black segment at the bottom left or top right has length 0.292*cos(theta), which comes out slightly less than 0.292.
Sorry, I got that wrong. The length of the referenced black segment is actually 1.283 - (.991*cos(theta)), which is slightly *larger* than 0.292, which is why your answer is coming out slightly higher than correct.
Curious, source and purpose of the problem?
A potential upcoming build.
 
Sorry, I got that wrong. The length of the referenced black segment is actually 1.283 - (.991*cos(theta)), which is slightly *larger* than 0.292, which is why your answer is coming out slightly higher than correct.

A potential upcoming build.
You have most definitely piqued my interest.

And now I have to go figure out if and how I went wrong. It will drive me to distraction until I do. 😅😂😅😂
 
And now I have to go figure out if and how I went wrong. It will drive me to distraction until I do. 😅😂😅😂
Just focus on that one value, how you got 0.292, despite the angle.

EDIT: LOL I got it wrong again. Third try:
1638979828038.png
Using the known correct value of theta, the short segment works out to 0.287, although you could not calculate this until you had correctly calculated theta.
 
Last edited:
Just focus on that one value, how you got 0.292, despite the angle.

EDIT: LOL I got it wrong again. Third try:
View attachment 493086
Using the known correct value of theta, the short segment works out to 0.287, although you could not calculate this until you had correctly calculated theta.

Yep, I see it now. I was just about to draw it out with extraneous info excluded.
Absolutely obvious and and a little embarrassing that I let it get by me. I am clearly rusty. :)

Thank you for your patience.... :cool::D
 
Back
Top